Course Content
All Previous Years Krok 2 Papers with Explanations
About Lesson

Question From (1 To 50 )

1. A 27-year-old man received an electrical injury, when he touched an exposed electrical wire with his hand, after which he developed circulatory and respiratory arrest. Resuscitation measures restored his cardiac activity after 5 minutes. What complications are possible in a few hours or even days after the electrical injury?

A. Circulatory arrest

B. Acute renal failure

C. Respiratory arrest

D. Pulmonary edema

E. Acute liver failure


Answer: Circulatory arrest

Explanation

It is important to note that electrical injuries can cause a wide range of complications, depending on the severity and duration of the electrical shock. In this case, the patient experienced circulatory and respiratory arrest immediately following the electrical injury, but was successfully resuscitated.  

One of the potential complications that can occur after an electrical injury is delayed circulatory arrest, which can occur several hours or even days after the initial injury. This can happen due to damage to the heart, blood vessels, or nervous system, and can result in cardiac arrest, shock, or other cardiovascular complications.  

Other possible complications of electrical injuries include acute renal failure, respiratory arrest, pulmonary edema, and acute liver failure, depending on the extent of organ damage. Therefore, it is important for the patient to receive appropriate monitoring and management to prevent and treat any potential complications.


2. Survey X-ray of the patient’s abdominal cavity has shown several hemispherical areas of lucency, located above welldefined horizontal levels. What might be the cause of such an X-ray presentation?

A. Intestinal obstruction

B. Cancer of the large intestine

C. Meteorism

D. Perforated ulcer

E. —


Answer:Intestinal obstruction

Explanation

The hemispherical areas of lucency seen on an abdominal X-ray, located above well-defined horizontal levels, are typically indicative of air-filled dilated loops of bowel, which can be a sign of intestinal obstruction.  

Intestinal obstruction occurs when there is a blockage in the small or large intestine that prevents the normal passage of food and fluids through the digestive system. This can lead to a buildup of gas and fluid in the blocked area, causing the bowel to become distended and resulting in the characteristic appearance of air-filled dilated loops of bowel on an X-ray image.  

Other potential causes of abdominal X-ray findings include cancer of the large intestine, perforated ulcer, and meteorism (excessive gas in the intestines), but these conditions would typically have different X-ray appearances than those seen in intestinal obstruction. Therefore, based on the X-ray findings described in the question, intestinal obstruction is the most likely cause of the presentation.


3. A patient has visited the doctor at a rural outpatient clinic twice during the calendar year (in March and in November). In both cases, he was diagnosed with acute respiratory viral infection. How to fill in the statistical forms for registration of final (clarified) diagnoses, to account for these morbidity cases?

A. For each case separately with the (+) sign

B. For each case separately with the (–) sign

C. For the first case with the (+) sign, for the second case with the (–) sign

D. The forms are not filled for either of these cases

E. For the first case with the (+) sign, for the second case no form is necessary


Answer:  For each case separately with the (+)sign

Explanation

In order to accurately track and register morbidity cases, each case should be recorded separately with the appropriate diagnosis code. In this case, the patient visited the doctor twice during the year with a diagnosis of acute respiratory viral infection. Therefore, each case should be recorded separately with the corresponding diagnosis code for acute respiratory viral infection.  

The (+) sign may indicate a positive diagnosis, while the (-) sign may indicate a negative diagnosis or a resolution of the condition. However, since the patient was diagnosed with acute respiratory viral infection on both visits, the appropriate designation for each case would be a (+) sign.   Therefore, the answer is: For each case separately with the (+) sign.


4. A 55-year-old man is complaining of general weakness, decreased fluid excretion, and an aching pain in his heart. For the last 15 years he has been suffering from chronic pyelonephritis and undergoing an inpatient treatment. His skin is dry and has a yellowish tinge. His pulse is 80/min., rhythmic, blood pressure – 100/70 mm Hg. Cardiac auscultation detects muffled heart sounds and a friction rub in the pericardium. Laboratory results: creatinine – 1.1 mmol/L, glomerular filtration – 5 mL/min. What treatment is indicated for this patient?

A. Hemodialysis

B. Diuretics

C. Rheopolyglucin (Dextran)

D. Xylit, Sorbitol

E. Antibiotics


Answer: Hemodialysis

Explanation

The patient in this case is presenting with symptoms of general weakness, decreased fluid excretion, and aching pain in the heart, along with a history of chronic pyelonephritis and inpatient treatment. The laboratory results also show low glomerular filtration rate (GFR) and elevated creatinine levels, indicating impaired kidney function. The presence of muffled heart sounds and a pericardial friction rub suggests the possibility of pericarditis, which can occur as a complication of chronic kidney disease.  

Given these findings, the most appropriate treatment for this patient would be hemodialysis. Hemodialysis is a medical procedure that filters waste and excess fluids from the blood when the kidneys are no longer able to perform this function adequately. This treatment is indicated for patients with advanced chronic kidney disease who have significant impairment of kidney function, as well as for those who may be experiencing complications such as fluid overload or electrolyte imbalances.  

Diuretics may be used to help manage fluid overload, but they may not be sufficient to address the underlying kidney dysfunction. Rheopolyglucin (Dextran) and Xylit, Sorbitol are not indicated for this patient’s condition, and antibiotics may not be necessary unless there is evidence of infection. Therefore, the most appropriate treatment for this patient would be hemodialysis.


5. After lifting a heavy object, a 41-yearold man felt a sharp pain in his chest on the right and developed progressing shortness of breath. The man’s condition is severe. His lips and mucosa are cyanotic, respiratory rate – 28/min., pulse – 122/min. Over the right half of the chest, percussion shows tympanitis, while auscultation detects sharply weakened breathing. The II heart sound is accentuated over the pulmonary artery. His blood pressure is 80/40 mm Hg. What primary emergency measure must be taken at the prehospital stage in this case?

A. Aspiration of air from the pleural cavity

B. Administration of euphyllin (aminophylline)

C. Call for a cardiology team

D. Administration of adrenaline

E. Oxygen inhalation


Answer: Aspiration of air from the pleural cavity

Explanation

The patient in this case has a history of lifting a heavy object and is now presenting with severe chest pain, shortness of breath, and cyanosis, along with physical exam findings suggestive of a possible pneumothorax. Given the severity of the patient’s condition, the primary emergency measure that should be taken at the prehospital stage is aspiration of air from the pleural cavity.  

Aspiration of air from the pleural cavity, also known as needle decompression, is a procedure that involves inserting a needle through the chest wall into the pleural space to remove air or other gases that may be causing the patient’s symptoms.

This procedure is typically performed in cases of tension pneumothorax, which can occur when air accumulates in the pleural space and causes a buildup of pressure that can compress the lung and other thoracic structures.   Administering euphyllin, calling for a cardiology team, or administering adrenaline may not be effective in treating the patient’s symptoms in this case.

Oxygen inhalation may provide some relief to the patient’s respiratory distress, but it is not the primary emergency measure that should be taken in this situation. Therefore, the most appropriate emergency measure for this patient at the prehospital stage would be aspiration of air from the pleural cavity.


6. For a month a 60-year-old patient had short-term episodes of decreased strength in his limbs on the left. Later, after waking up in the morning, he developed a persistent weakness in his extremities. His blood pressure is 140/90 mm Hg. He is conscious and has central paresis of VII and XII pairs of cranial nerves on the left. On the same side, he presents with central hemiparesis and hemihyperesthesia. What group of drugs must be chosen for the differentiated treatment of this patient?

A. Anticoagulants

B. Hypotensive agents

C. Diuretics

D. Corticosteroids

E. Hemostatics


Answer: Anticoagulants

Explanation

The patient in this case is presenting with symptoms of central paresis of the VII and XII cranial nerves on the left, along with central hemiparesis and hemihyperesthesia on the same side. These symptoms are suggestive of a possible stroke, which may be caused by a blood clot blocking blood flow to the brain.  

Given these findings, the most appropriate group of drugs for the differentiated treatment of this patient would be anticoagulants. Anticoagulants are medications that help to prevent the formation of blood clots, which can reduce the risk of stroke and other cardiovascular complications.   Hypotensive agents and diuretics may be used to manage hypertension, which is a risk factor for stroke, but they may not be sufficient to address the underlying blood clot.

Corticosteroids and hemostatics are not indicated for the treatment of stroke and may even be contraindicated in some cases.   Therefore, the most appropriate group of drugs for the differentiated treatment of this patient would be anticoagulants. However, the specific choice of anticoagulant medication and dosing should be individualized based on the patient’s clinical features and medical history, and should be made in consultation with a healthcare provider.


7. A 45-year-old woman with a normal body weight was diagnosed with diabetes mellitus for the first time. Attempts to correct his blood glucose levels by means of a diet were unsuccessful. Her 24- hour glycemia varies between 10 and 15 mmol/L. Which medicine would be optimal in this case?

A. Sulfanilamide hypoglycemic drugs

B. Biguanides combined with sulfanilamide hypoglycemic drugs

C. Insulin

D. Insulin combined with sulfanilamide hypoglycemic drugs

E. Biguanides


Answer:  Sulfanilamide hypoglycemic drugs

Explanation

The patient in this case has been diagnosed with diabetes mellitus for the first time and has been unable to achieve adequate blood glucose control with dietary changes alone. Her 24-hour glycemia remains high, between 10 and 15 mmol/L. Given these findings, the optimal medication for this patient would be sulfanilamide hypoglycemic drugs.  

Sulfanilamide hypoglycemic drugs, also known as sulfonylureas, are medications that stimulate the pancreas to secrete more insulin, which helps to lower blood glucose levels. They are commonly used in the treatment of type 2 diabetes mellitus, particularly in cases where lifestyle changes such as diet and exercise have been unsuccessful in controlling blood glucose levels.  

While biguanides such as metformin may also be effective in lowering blood glucose levels, they are typically considered as second-line therapy after sulfanilamide hypoglycemic drugs. Insulin therapy may be necessary in some cases of diabetes mellitus, but it is typically reserved for more severe cases or for patients who are unable to tolerate oral medications.  

Therefore, the most appropriate medication for this patient would be sulfanilamide hypoglycemic drugs. However, the specific choice of medication and dosing should be individualized based on the patient’s clinical features and medical history, and should be made in consultation with a healthcare provider.


8. Fluorography of a 45-year-old patient detects a few foci of low intensity with blurred margins on the apex of his right lung. This sign is observed for the first time. Such condition causes him no discomfort. He has a many-year history of smoking. Objectively, percussion produces a pulmonary sound above the lungs, the respiration is vesicular, auscultation detects no wheezing. Blood test findings are normal. Make the right diagnosis:

A. Focal pulmonary tuberculosis

B. Peripheral lung cancer

C. Bronchopneumonia

D. Disseminated pulmonary tuberculosis

E. Eosinophilic pneumonia


Answer: Focal pulmonary tuberculosis

Explanation

The findings in this case suggest a possible diagnosis of focal pulmonary tuberculosis. The presence of low-intensity foci with blurred margins on the apex of the right lung, along with a history of smoking and no respiratory discomfort, are consistent with the radiographic and clinical presentation of tuberculosis.  

Focal pulmonary tuberculosis is a form of tuberculosis that is characterized by the presence of small, isolated lesions in the lungs. It may be asymptomatic or may cause mild symptoms such as cough, low-grade fever, and fatigue. Smoking is a risk factor for developing tuberculosis, as it can weaken the immune system and make individuals more susceptible to infection.  

Peripheral lung cancer is less likely given the absence of any specific signs or symptoms suggestive of lung cancer, such as coughing up blood, weight loss, or persistent cough. Bronchopneumonia is also less likely, as it typically presents with more widespread infiltrates in the lungs and is associated with fever, cough, and shortness of breath.  

Disseminated pulmonary tuberculosis is characterized by the spread of tuberculosis infection beyond the lungs to other parts of the body, and is less likely given the localized nature of the foci. Eosinophilic pneumonia is also less likely, as it typically presents with eosinophilia in the blood and symptoms such as cough, fever, and shortness of breath.  

Therefore, based on the radiographic and clinical findings, the most likely diagnosis in this case is focal pulmonary tuberculosis. Further testing, such as sputum culture and chest CT scan, may be necessary to confirm the diagnosis and to guide appropriate treatment.


9. A 54-year-old patient has been suffering from femoral osteomyelitis for over 20 years. In the last month she developed gradually progressing leg edemas. In the urine: proteinuria – 6.6 g/L. In the blood: dysproteinemia in the form of hypoalbuminemia, increased α2- and γglobulin levels, ESR – 50 mm/hour. Make the right diagnosis:

A. Secondary renal amyloidosis

B. Acute glomerulonephritis

C. Systemic lupus erythematosus

D. Chronic glomerulonephritis

E. Multiple myeloma


Answer:  Secondary renal amyloidosis

Explanation

The patient in this case has a long-standing history of femoral osteomyelitis and is now presenting with gradually progressing leg edema, proteinuria, hypoalbuminemia, and dysproteinemia with increased α2- and γ-globulin levels. These findings are consistent with the diagnosis of secondary renal amyloidosis.  

Secondary renal amyloidosis is a condition in which abnormal protein deposits called amyloid accumulate in the kidneys, leading to kidney dysfunction. It can occur as a complication of chronic inflammatory or infectious conditions such as osteomyelitis, and is characterized by proteinuria, hypoalbuminemia, and dysproteinemia.  

Acute glomerulonephritis and chronic glomerulonephritis are also types of kidney disease, but they typically present with different clinical and laboratory findings, such as hematuria, hypertension, and elevated serum creatinine levels. Systemic lupus erythematosus is an autoimmune disease that can affect multiple organ systems, including the kidneys, but it typically presents with a combination of clinical and laboratory findings such as rash, joint pain, and positive antinuclear antibody (ANA) test.

Multiple myeloma is a type of cancer that affects the plasma cells in the bone marrow, and typically presents with abnormal blood counts, bone pain, and other symptoms.   Therefore, based on the clinical and laboratory findings in this case, the most likely diagnosis is secondary renal amyloidosis. Further testing, such as kidney biopsy, may be necessary to confirm the diagnosis and to guide appropriate treatment.


10. A 28-year-old man was hospitalized on the 9th day of illness with complaints of fever of 39oC, headache, general weakness, constipation, and disturbed sleep. On the skin of his abdomen there were isolated roseolas, his pulse is 78/min., the liver is enlarged by 2 cm. Make the right diagnosis:

A. Typhoid fever

B. Brucellosis

C. Leptospirosis

D. Sepsis

E. Epidemic typhus


Answer: Typhoid fever

Explanation

The clinical presentation of fever, headache, constipation, disturbed sleep, and isolated roseolas on the abdomen, along with an enlarged liver, is suggestive of typhoid fever.   Typhoid fever is a bacterial infection caused by the bacterium Salmonella typhi, and is characterized by fever, headache, abdominal pain, constipation (or diarrhea), and a rash of rose-colored spots.

The disease is transmitted through contaminated food or water, and is more common in areas with poor sanitation.   Brucellosis, leptospirosis, and epidemic typhus can also cause fever and other similar symptoms, but they typically present with additional signs or symptoms that are not mentioned in this case.

Sepsis is a severe, life-threatening condition that can result from any infection, but is typically associated with more severe symptoms such as high fever, low blood pressure, and organ dysfunction.   Therefore, based on the clinical presentation and history of the patient, the most likely diagnosis is typhoid fever. Further testing, such as blood cultures or stool cultures, may be necessary to confirm the diagnosis and to guide appropriate treatment.


11. A well was dug on a rural land plot. It is located at the distance of 20 meters from the house, 10 meters from the toilet, and 15 meters from the neighbor’s house. What is the minimum permissible distance between the well and the source of possible water pollution, according to the sanitary norms?

A. 30 meters

B. 25 meters

C. 20 meters

D. 15 meters

E. 10 meters


Answer: 30 meters

Explanation

According to sanitary norms, the minimum permissible distance between a well and potential sources of water pollution depends on the type of source. In this case, the well is located 20 meters from the house, 10 meters from the toilet, and 15 meters from the neighbor’s house.  

The minimum permissible distance between the well and the toilet is generally at least 30 meters to prevent contamination of the well water by fecal matter. The well should also be located at least 15 meters away from any other potential sources of pollution, such as septic tanks, livestock pens, or chemical storage areas.  

Therefore, since the well in this case is only 10 meters away from the toilet and 15 meters away from the neighbor’s house, it does not meet the minimum permissible distance requirements for these potential sources of pollution. The minimum permissible distance for the toilet is at least 30 meters, so the correct answer is A. 30 meters.


12. The ambulance has brought in the victim of an accident, who has fallen from a height. This patient has clinical signs of multiple fractures in both legs. The patient’s condition is severe; the patient is conscious, but mentally sluggish; the skin is pale gray; the forehead is covered in a cold sweat. The patient’s breathing is shallow, with respiratory rate of 30/min., blood pressure – 80/60 mm Hg, pulse – 120/min., shallow. What complication has occurred in this person?

A. Traumatic shock, degree 2

B. Traumatic shock, degree 1

C. Traumatic shock, degree 4

D. Syncope

E. Traumatic shock, degree 3


Answer:  Traumatic shock, degree 2

Explanation

The patient in this case has been involved in an accident and has multiple fractures in both legs. The patient is conscious but mentally sluggish, with a pale gray skin color, cold sweat on the forehead, shallow breathing, and a rapid, shallow pulse and blood pressure of 80/60 mm Hg.

These symptoms suggest the presence of traumatic shock, likely degree 2.   Traumatic shock occurs when there is a significant loss of blood volume or fluid from the body, which can occur in cases of severe trauma such as multiple fractures or hemorrhage. The symptoms of traumatic shock include a decrease in blood pressure, rapid and shallow breathing, rapid pulse, and pale, cool, clammy skin.


The severity of traumatic shock is typically classified into four degrees, with degree 1 being the mildest and degree 4 being the most severe.   In this case, the patient’s blood pressure and mental status suggest that the shock is not yet severe enough to be classified as degree 3 or 4. However, the patient’s pulse rate, breathing rate, and skin color suggest that the shock is moderate, which would be consistent with degree 2.  

Syncope, or fainting, is a loss of consciousness that is typically caused by a sudden drop in blood pressure or a decrease in blood flow to the brain. While syncope can occur as a result of traumatic shock, the patient’s symptoms in this case do not suggest a sudden loss of consciousness.  

Therefore, based on the clinical presentation of the patient, the most likely complication is traumatic shock, degree 2. Immediate treatment is necessary to stabilize the patient’s vital signs and prevent progression to more severe degrees of shock.


13. A 30-year-old patient after an emotional upset developed tonic seizure that later gave place to clonic seizures. This condition was accompanied by loss of consciousness, foaming at the mouth, and involuntary urination. No focal signs were detected. Blood pressure is 120/60 mm Hg. Make the right diagnosis:

A. Epilepsy

B. Syncope

C. Neurasthenia

D. Eclampsia

E. Stroke


Answer:Epilepsy

Explanation

The clinical presentation of tonic-clonic seizures, loss of consciousness, foaming at the mouth, and involuntary urination in a young patient after an emotional upset is highly suggestive of epilepsy.   Epilepsy is a neurological disorder characterized by recurrent seizures, which can present in various forms including tonic-clonic seizures.

Seizures typically involve abnormal electrical activity in the brain, which can cause a wide range of symptoms including loss of consciousness, muscle spasms, and sensory disturbances.   Syncope, or fainting, typically does not involve convulsive movements or foaming at the mouth, and is often caused by a sudden drop in blood pressure or a decrease in blood flow to the brain.

Neurasthenia is a term that has been used to describe a collection of symptoms related to fatigue and nervous exhaustion, but it is not a specific diagnosis and is not typically associated with seizures. Eclampsia is a complication of pregnancy that can cause seizures, but the patient in this case is not pregnant.

Stroke typically presents with focal neurological signs such as weakness or numbness on one side of the body, which are not present in this case.   Therefore, based on the clinical presentation, the most likely diagnosis is epilepsy. Further testing, such as electroencephalography (EEG), may be necessary to confirm the diagnosis and to guide appropriate treatment.


14. A full-term delivery has resulted in the birth of a live full-term girl without asphyxia. The baby is inert, her skin is pale with an icteric tinge, she has no edemas. The abdomen is soft, the liver and spleen are enlarged. The blood tests show that the mother’s blood type is A(II) Rh(–), while the child’s blood type is A(II) Rh(+). Make the right diagnosis:

A. Hemolytic disease of the newborn

B. Intracranial birth injury

C. Physiological jaundice

D. Maldevelopment of parenchymal organs

E. Disturbed cerebral circulation


Answer: Hemolytic disease of the newborn

Explanation

The clinical presentation of a full-term baby with pale skin, an icteric tinge, enlarged liver and spleen, and a positive Rh factor in the blood type, suggests the diagnosis of hemolytic disease of the newborn.   Hemolytic disease of the newborn occurs when there is an incompatibility between the mother’s blood type and the baby’s blood type, which causes the mother’s immune system to attack the baby’s red blood cells.

This can lead to hemolysis, or breakdown of red blood cells, which can cause anemia, jaundice, and enlargement of the liver and spleen.   In this case, the baby’s blood type is A(II) Rh(+), while the mother’s blood type is A(II) Rh(-).

This Rh incompatibility can cause the mother’s immune system to produce antibodies that attack the baby’s red blood cells, leading to hemolysis and the clinical symptoms described.   Intracranial birth injury would typically present with neurological symptoms such as seizures, abnormal movements, or altered consciousness.

Physiological jaundice is a common condition in newborns that typically presents with mild jaundice and no other significant symptoms. Maldevelopment of parenchymal organs or disturbed cerebral circulation are unlikely to be the cause of the clinical presentation described.  

Therefore, based on the clinical presentation and the blood type incompatibility, the most likely diagnosis is hemolytic disease of the newborn. Appropriate treatment, such as phototherapy or exchange transfusion, may be necessary to manage the symptoms and prevent further complications.


15. The region in the Carpathian foothills is characterized with constant high atmospheric humidity (over 80%). In the cold season, while the air temperature is moderately low, the population of this region feels extreme cold. What type of heat transfer increases in such conditions?

A. Convection

B. – C. Evaporation

D. Radiation

E. Conduction


Answer:  Convection

Explanation

The region in the Carpathian foothills is characterized by high atmospheric humidity and moderate low air temperature in the cold season, which can result in the population feeling extremely cold. In such conditions, the type of heat transfer that increases is convection.  

Convection is the transfer of heat through a fluid, such as air or water, due to the movement of the fluid. In this case, the high atmospheric humidity can increase the moisture content in the air, making it more humid and reducing the rate of evaporation. This can result in the air feeling heavier and more difficult to move, thus reducing the rate of heat transfer by convection.  

On the other hand, the reduced rate of evaporation also reduces the rate of heat transfer by evaporation, which is another important mechanism for dissipating heat from the body. Radiation and conduction are less significant in this scenario, as they are not strongly influenced by the properties of the surrounding air.   Therefore, based on the given conditions, the type of heat transfer that increases in such conditions is convection.


16. A 58-year-old patient complains of 922521I1 3 an inguinal tumor that increases during straining and coughing. There is a pulsating tumor-like formation located below and laterally to the Poupart’s ligament. The formation does not diminish during palpation. Make the right diagnosis:

A. Femoral aneurysm

B. Femoral hernia

C. Neurinoma of the femoral nerve

D. Ectopic testicle

E. Inguinal hernia


Answer: Femoral aneurysm

Explanation

The clinical presentation of a pulsating tumor-like formation located below and laterally to the Poupart’s ligament, which increases during straining and coughing, suggests the diagnosis of femoral aneurysm.   Femoral aneurysm is a bulging or ballooning of the wall of the femoral artery, which is located in the groin area.

The aneurysm can cause a pulsating mass in the groin, which may be more noticeable during straining or coughing. Unlike hernias, the mass does not typically diminish with palpation. Femoral aneurysms are more common in older adults and can be associated with other vascular disease risk factors.  

Femoral hernias can also present as a bulge in the groin, but they are typically more medial and do not have a pulsatile quality. Neurinomas of the femoral nerve are rare and would not present as a pulsatile mass in the groin. Ectopic testicles are typically located outside of the normal scrotal position and would not present as a mass in the groin.

Inguinal hernias are also characterized by a bulge in the groin, but they are typically more medial and are not pulsatile.   Therefore, based on the clinical presentation, the most likely diagnosis is femoral aneurysm. Further testing, such as ultrasound or computed tomography (CT) angiography, may be necessary to confirm the diagnosis and to guide appropriate treatment.


17. A variety of lighting fixtures have been used to illuminate classrooms. What kind of lighting fixtures creates the most hygienically acceptable lighting?

A. Lighting fixtures that provide reflected lighting

B. Lighting fixtures that provide direct lighting

C. Lighting fixtures that provide scattered lighting

D. Lighting fixtures that provide semireflected lighting

E. Lighting fixtures that provide combined Lighting


Answer:  Lighting fixtures that provide reflected lighting

Explanation

lighting   Lighting fixtures that provide reflected lighting create the most hygienically acceptable lighting in classrooms. Reflected lighting fixtures direct light towards the ceiling, walls, and other surfaces, which then reflect the light back into the room. This creates a diffuse and uniform illumination that minimizes glare and shadows, which can reduce eye strain and improve visibility.  

In addition, reflected lighting fixtures can help to reduce the accumulation of dust and other airborne particles, which can contribute to poor air quality and aggravate respiratory conditions.

By directing light towards surfaces rather than directly into the eyes, reflected lighting fixtures can minimize the disturbance of dust and other particles in the air.   Direct lighting fixtures, on the other hand, can create glare and shadows, which can be uncomfortable and distracting for students and teachers.

Scattered lighting fixtures can also create uneven illumination and shadows, which can reduce visibility and make it more difficult to read or see objects clearly.   Semireflected lighting fixtures and combined lighting fixtures can provide some benefits of both reflected and direct lighting, but they may not be as effective at minimizing glare and shadows as reflected lighting fixtures.  

Therefore, based on the goal of creating hygienically acceptable lighting in classrooms, lighting fixtures that provide reflected lighting are the most appropriate choice.


18. A victim of a car accident received multiple fractures of the limbs and pelvic bones. He has a history of hemophilia A. Objectively, hematomas form at the sites of injury. The patient’s condition is deteriorating. The blood pressure is 90/50 mm Hg. What is the optimal combination of infusion agents that should be given to the patient for treatment after polyglucin (dextran) and saline solutions are administered?

A. Cryoprecipitate, packed erythrocytes

B. Packed erythrocytes, fresh frozen plasma

C. Fresh frozen plasma, albumin

D. Cryoprecipitate, glucose

E. Packed erythrocytes


Answer: Cryoprecipitate, packed erythrocytes

Explanation

The patient in this scenario has multiple fractures and a history of hemophilia A, and is experiencing hematomas at the sites of injury. The patient’s condition is deteriorating, and their blood pressure is low. In order to manage the patient’s condition, the optimal combination of infusion agents would be cryoprecipitate and packed erythrocytes.  

Cryoprecipitate is a blood product that contains concentrated levels of clotting factors, including factor VIII which is deficient in hemophilia A. By administering cryoprecipitate, the patient’s clotting ability can be improved, which may help to prevent or reduce bleeding at the sites of injury. Packed erythrocytes are also necessary to replace the patient’s lost blood volume and improve oxygen delivery to the tissues.  

Fresh frozen plasma contains clotting factors and may be useful in managing bleeding in patients with hemophilia A, but it is less concentrated than cryoprecipitate and may not be as effective in improving clotting ability. Albumin is a protein that can be used to manage fluid balance in some patients, but it is not indicated in this scenario.  

Glucose alone would not be appropriate for managing the patient’s condition, as it does not address the underlying issues of blood loss and impaired clotting. Packed erythrocytes alone may help to replace lost blood volume, but cryoprecipitate is necessary to improve clotting ability.   Therefore, based on the clinical presentation and the patient’s history, the optimal combination of infusion agents for this patient would be cryoprecipitate and packed erythrocytes.


19. A 40-year-old woman, gravida 6, para 3, at the 40 weeks of her pathological pregnancy (threatened miscarriage, type I gestosis of the second half of her pregnancy), gave birth to a girl with asphyxia. The baby’s condition is severe, the weight is 2 kg, there are signs of immaturity and hydrocephalus. The baby’s skin is pale, icteric, and has acrocyanosis. Heart sounds are muffled, there is a harsh systolic murmur at all points of auscultation. Abdomen is enlarged, the liver is +3 cm. The urine is saturated, the stool is light-colored. An ophthalmologist has detected chorioretinitis in the baby. Make the right diagnosis:

A. Congenital toxoplasmosis

B. Hemolytic disease of the newborn

C. Sepsis

D. Congenital heart defect

E. Congenital hepatitis


Answer: Congenital toxoplasmosis

Explanation

The clinical presentation of a newborn with signs of immaturity, hydrocephalus, pale and icteric skin with acrocyanosis, muffled heart sounds with a harsh systolic murmur, an enlarged abdomen with an enlarged liver, saturated urine, light-colored stool, and chorioretinitis suggests the diagnosis of congenital toxoplasmosis.  

Congenital toxoplasmosis is a parasitic infection that can be transmitted from an infected mother to her developing fetus during pregnancy. The infection can cause a range of symptoms in the newborn, including neurological problems, eye infections, hepatosplenomegaly, and jaundice.

The presence of chorioretinitis in the newborn is a hallmark feature of congenital toxoplasmosis.   Hemolytic disease of the newborn can cause anemia, jaundice, and enlarged liver and spleen, but it typically does not cause neurological symptoms or chorioretinitis. Sepsis is a serious bacterial infection that can cause a range of symptoms, including fever, respiratory distress, and low blood pressure, but it would not typically cause the specific constellation of symptoms described in this scenario.

Congenital heart defects can cause heart murmurs, but they would not typically cause the other symptoms described in this scenario. Congenital hepatitis can cause jaundice and liver damage, but it would not typically cause the neurological symptoms or chorioretinitis described in this scenario.  

Therefore, based on the clinical presentation and the presence of chorioretinitis, the most likely diagnosis is congenital toxoplasmosis. Prompt diagnosis and treatment with appropriate antiparasitic medications can improve outcomes for affected newborns.


20. A woman at 12 weeks of her pregnancy developed a pain in her lower abdomen and mild bloody Discharge, which occurred with no apparent reason. Vaginal examination shows well-formed cervix with closed external orifice. The body of the uterus is enlarged according to her term of pregnancy. Make the right diagnosis:

A. Threatened miscarriage

B. Anembryonic pregnancy

C. Ongoing miscarriage

D. Imminent miscarriage

E. Incomplete miscarriage


Answer: Threatened miscarriage

Explanation

The clinical presentation of a woman at 12 weeks of pregnancy with lower abdominal pain and mild vaginal bleeding, with a well-formed cervix with a closed external orifice and an enlarged uterine body according to the term of pregnancy, suggests the diagnosis of threatened miscarriage.  

Threatened miscarriage is a common occurrence during early pregnancy and is typically defined as vaginal bleeding and/or abdominal pain before the 20th week of gestation, with a closed cervix and no evidence of fetal demise. The bleeding can be light or heavy, and may be accompanied by cramping or pain. In many cases, the pregnancy will continue without further complications.  

Anembryonic pregnancy, also known as a blighted ovum, occurs when a gestational sac develops in the uterus, but no embryo forms or the embryo stops developing early on. This would typically be identified by ultrasound, which would show an empty gestational sac.  

Ongoing miscarriage, also known as incomplete miscarriage, occurs when the pregnancy has ended but not all of the products of conception have been expelled from the uterus. This would typically be identified by ultrasound or clinical examination, which would show an open cervix and evidence of fetal demise.  

Imminent miscarriage, also known as inevitable miscarriage, occurs when the cervix is dilated and there is evidence of bleeding or rupture of membranes. This would typically require urgent medical attention.   Incomplete miscarriage, also known as ongoing miscarriage, occurs when the pregnancy has ended but not all of the products of conception have been expelled from the uterus.

This would typically be identified by ultrasound or clinical examination, which would show an open cervix and evidence of fetal demise.   Therefore, based on the clinical presentation and examination findings, the most likely diagnosis is threatened miscarriage. Close monitoring and follow-up with healthcare providers is important to manage the condition and monitor for any complications.


21. In the demographic structure of a region, the people aged from 0 to 14 make up 31% of the population, while the 50+ age group makes up 20% of the population. What population structure characterizes such demographic situation most accurately?

A. Expansive population

B. Stationary population

C. Constrictive population

D. Emigration of the population

E. Migration of the population


Answer:  Expansive population

Explanation

The demographic structure described, with 31% of the population aged 0-14 and 20% aged 50 and above, characterizes an expansive population.   An expansive population structure is characterized by a high proportion of young people (under age 15) and a relatively low proportion of older people (typically defined as age 65 and above).

This type of population structure is often associated with developing countries and regions, where there is high fertility and birth rates, and low life expectancy due to poor healthcare and living conditions. The high proportion of young people in an expansive population structure can create challenges for healthcare, education, and employment opportunities in the region.  

In contrast, a stationary population structure is characterized by roughly equal proportions of young and middle-aged people, with a smaller proportion of older people. This type of population structure is often associated with developed countries where birth and death rates are low and life expectancy is high.  

A constrictive population structure is characterized by a low proportion of young people and a higher proportion of older people. This type of population structure can occur in developed countries with low fertility rates and high life expectancy.  

Emigration and migration refer to the movement of people across borders, and are not characteristics of population structures.   Therefore, based on the proportions of different age groups described, the most accurate characterization of this demographic situation is an expansive population.


22. A 13-year-old boy is complaining of a dry cough and shortness of breath. The onset of the disease was one year ago. He has brief asphyxia attacks that occur 1–2 times per month. Objectively, the child is anxious and has expiratory dyspnea, his skin is pale, his nasolabial triangle is cyanotic. His respiratory rate is 48/min. Percussion produces a banbox resonance over the lungs; auscultation detects weakened breathing with bilateral dry wheezing. Forced expiratory volume is 80% of the normal. What medicine should be prescribed to this boy?

A. Salbutamol

B. Euphyllin (Aminophylline)

C. Prednisolone

D. Indomethacin

E. Suprastin (Chloropyramine)


Answer: Salbutamol

Explanation

The clinical presentation of a 13-year-old boy with a dry cough, shortness of breath, asphyxia attacks, expiratory dyspnea, cyanosis, tachypnea, banbox resonance on percussion, weakened breathing with bilateral dry wheezing, and forced expiratory volume of 80% of normal is suggestive of asthma.

The most appropriate medication to prescribe for this patient is salbutamol. Salbutamol is a short-acting beta-2 agonist that acts by relaxing the smooth muscles of the airways, thus improving breathing and reducing symptoms such as shortness of breath and wheezing. It is commonly used as a rescue medication for the acute relief of asthma symptoms.

Euphyllin (Aminophylline) is a bronchodilator that is used less frequently today compared to salbutamol or other beta-2 agonists due to its potential side effects and narrow therapeutic window. Prednisolone is a corticosteroid that can be used to reduce inflammation and improve symptoms in patients with asthma, but it is typically used as a maintenance medication rather than a rescue medication.

Indomethacin is a nonsteroidal anti-inflammatory drug (NSAID) that is used to treat pain and inflammation, but it is not indicated for the treatment of asthma. Suprastin (Chloropyramine) is an antihistamine that can be used to reduce symptoms such as itching and redness caused by allergic reactions, but it is not typically used to treat asthma. Therefore, based on the clinical presentation and the most appropriate medication for managing asthma symptoms, the correct answer is salbutamol.


23. A patient with rheumatism presents with diastolic tremor of the chest wall («cat’s purr»), heart sound I that is amplified at the apex, diastolic murmur with presystolic amplification, a mitral opening sound, and an accent of the II 922521I1 4 heart sound over the pulmonary artery. What heart defect was observed in this patient?

A. Stenosis of the left atrioventricular orifice

B. Mitral valve insufficiency

C. Patent ductus arteriosus

D. Pulmonary artery stenosis

E. Aortic valve insufficiency


Answer:  Stenosis of the left atrioventricular orifice

Explanation

The clinical presentation of a patient with rheumatism who has a diastolic tremor of the chest wall, amplified heart sound I at the apex, diastolic murmur with presystolic amplification, a mitral opening sound, and an accent of the second heart sound over the pulmonary artery is suggestive of stenosis of the left atrioventricular orifice, also known as mitral stenosis.  

Mitral stenosis is a heart valve disorder in which the mitral valve, located between the left atrium and left ventricle, is narrowed or obstructed. This can lead to increased pressure in the left atrium, which can result in symptoms such as shortness of breath, fatigue, and chest pain.  

The diastolic tremor of the chest wall, also known as a “cat’s purr” or “tumor plop,” is a rare finding that can be heard in patients with mitral stenosis. The amplified heart sound I at the apex, diastolic murmur with presystolic amplification, and mitral opening sound are also characteristic features of mitral stenosis.

The accentuation of the second heart sound over the pulmonary artery is due to increased pressure in the right ventricle, which can occur as a result of mitral stenosis.   Mitral valve insufficiency, also known as mitral regurgitation, is a heart valve disorder in which the mitral valve does not close properly, allowing blood to flow backwards into the left atrium.

This would typically present with a systolic murmur rather than a diastolic murmur.   Patent ductus arteriosus is a congenital heart defect in which a blood vessel called the ductus arteriosus, which connects the pulmonary artery to the aorta during fetal development, fails to close after birth. This would typically present with a continuous murmur rather than a diastolic murmur.  

Pulmonary artery stenosis is a heart valve disorder in which the pulmonary valve, located between the right ventricle and pulmonary artery, is narrowed or obstructed. This would typically present with a systolic murmur rather than a diastolic murmur.  

Aortic valve insufficiency, also known as aortic regurgitation, is a heart valve disorder in which the aortic valve does not close properly, allowing blood to flow backwards into the left ventricle. This would typically present with a diastolic murmur, but it would not be associated with a diastolic tremor or mitral opening sound.   Therefore, based on the clinical presentation and characteristic features, the correct answer is stenosis of the left atrioventricular orifice, also known as mitral stenosis.


24. At a factory with harmful working conditions, a complex of measures was being taken to reduce the morbidity among the workers. For better effect, the factory doctor was separately monitoring a group of people, who fall ill frequently and for a long time. What type of morbidity requires distinguishing such a group of people?

A. Morbidity with temporary disability

B. Morbidity associated with acute

infections

C. General morbidity

D. Morbidity associated with major noncommunicable diseases

E. Hospitalized morbidity


Answer: Morbidity with temporary disability

Explanation

The group of people who fall ill frequently and for a long time should be distinguished based on morbidity with temporary disability.   Morbidity with temporary disability refers to illnesses and conditions that result in a temporary inability to work, such as respiratory infections, musculoskeletal disorders, and injuries.

This type of morbidity can have a significant impact on productivity and can lead to increased healthcare costs, absenteeism, and reduced quality of life.   In a factory with harmful working conditions, it is important to monitor workers who experience frequent and prolonged episodes of illness, as they may be at a higher risk of developing work-related illnesses or injuries.

By identifying and addressing the underlying causes of their illnesses, such as exposure to harmful substances or ergonomic issues, it may be possible to reduce the incidence of morbidity with temporary disability and improve the overall health and well-being of the workforce.   Morbidity associated with acute infections refers to illnesses caused by infectious agents, such as viruses or bacteria, that typically have a rapid onset and short duration. This type of morbidity is usually not associated with prolonged periods of disability.  

General morbidity refers to the overall incidence of illness or disease in a population, without distinguishing between different types of illnesses or disabilities.   Morbidity associated with major noncommunicable diseases refers to chronic conditions such as cardiovascular disease, cancer, and diabetes. This type of morbidity is not typically associated with temporary disability, but rather with long-term management and treatment.  

Hospitalized morbidity refers to illnesses or conditions that require hospitalization for treatment, and may include both acute and chronic conditions.   Therefore, based on the description of the factory and the group of workers being monitored, the type of morbidity that requires distinguishing such a group of people is morbidity with temporary disability.


25. A 38-year-old patient has episodes of paroxysmal hypertension that reaches 240/120 mm Hg and is accompanied by nausea, vomiting, tachycardia, and excessive sweating. Hyperglycemia is observed in the blood during such episodes. After the episode, profuse urination occurs. Renal sonography has shown a new formation adjacent to the upper pole of the right kidney that might belong to the adrenal gland. What laboratory test will help clarify the diagnosis?

A. Urinary excretion of catecholamines and vanillylmandelic acid

B. Blood levels of thyroxine and thyroidstimulating hormone

C. Blood renin levels

D. Blood levels of insulin and C-peptide

E. Glomerular filtration rate, measured using the endogenous creatinine clearance Rate


Answer: Urinary excretion of catecholamines and vanillylmandelic acid

Explanation

The clinical presentation of a 38-year-old patient with episodes of paroxysmal hypertension, nausea, vomiting, tachycardia, excessive sweating, hyperglycemia, and profuse urination, along with the presence of a new formation adjacent to the upper pole of the right kidney that might belong to the adrenal gland, is suggestive of a pheochromocytoma.

The laboratory test that will help clarify the diagnosis is urinary excretion of catecholamines and vanillylmandelic acid (VMA).   Pheochromocytoma is a rare tumor of the adrenal gland that secretes excessive amounts of catecholamines, such as epinephrine and norepinephrine. The tumor can cause episodic paroxysmal hypertension, which is characterized by sudden and severe increases in blood pressure accompanied by symptoms such as nausea, vomiting, tachycardia, and excessive sweating.

Hyperglycemia and profuse urination can also occur due to the effects of catecholamines on glucose metabolism and renal function.   Urinary excretion of catecholamines and VMA is a useful diagnostic test for pheochromocytoma. Elevated levels of catecholamines and VMA in the urine can indicate the presence of the tumor.

The test can be performed during or immediately after an episode of hypertension to increase the sensitivity of the results.   Blood levels of thyroxine and thyroid-stimulating hormone (TSH) are used to evaluate thyroid function and are not directly related to the diagnosis of pheochromocytoma.   Blood renin levels are used to evaluate the renin-angiotensin-aldosterone system and are not directly related to the diagnosis of pheochromocytoma.  

Blood levels of insulin and C-peptide are used to evaluate pancreatic function and are not directly related to the diagnosis of pheochromocytoma.   Glomerular filtration rate, measured using the endogenous creatinine clearance rate, is used to evaluate kidney function and is not directly related to the diagnosis of pheochromocytoma.   Therefore, based on the clinical presentation and the most appropriate laboratory test for diagnosing pheochromocytoma, the correct answer is urinary excretion of catecholamines and VMA.


26. During physical exertion, a man with a cerebral artery aneurysm suddenly developed severe headache and vomiting and fainted. The patient is agitated and wants to run somewhere. His pulse is 62/min., rhythmic, blood pressure – 140/90 mm Hg, the body temperature is 37.5oC. He has nuchal rigidity and positive Kernig’s sign. Focal neurological signs are absent. Make the provisional diagnosis:

A. Subarachnoid hemorrhage

B. Hypertensive crisis

C. Ischemic stroke

D. Meningitis

E. Brain hemorrhage


Answer:  Subarachnoid hemorrhage

Explanation

The clinical presentation of a man with a cerebral artery aneurysm who suddenly developed severe headache and vomiting, fainted, and is now agitated with nuchal rigidity and positive Kernig’s sign, is highly suggestive of subarachnoid hemorrhage.   Subarachnoid hemorrhage is a type of bleeding that occurs in the space between the brain and the thin tissues that cover the brain, called the arachnoid membrane.

It is commonly caused by the rupture of an aneurysm, which is a weakened and bulging area in a blood vessel in the brain. The sudden onset of severe headache, vomiting, and loss of consciousness are classic symptoms of subarachnoid hemorrhage, which can be exacerbated by physical exertion.  

Nuchal rigidity, or stiffness in the neck, is a common sign of subarachnoid hemorrhage, as the bleeding can irritate and inflame the membranes surrounding the brain and spinal cord. Positive Kernig’s sign, which is the resistance to knee extension when the hip is flexed, is also a sign of meningeal irritation, which can occur with subarachnoid hemorrhage.  

Hypertensive crisis, ischemic stroke, and brain hemorrhage can also cause sudden onset of severe headache and vomiting, but the presence of nuchal rigidity and positive Kernig’s sign suggest subarachnoid hemorrhage as the most likely diagnosis.   Meningitis can also present with headache, vomiting, and nuchal rigidity, but typically has a more gradual onset and may have additional symptoms such as fever, photophobia, and altered mental status.

Focal neurological signs, such as weakness or numbness on one side of the body, would be more indicative of ischemic stroke or brain hemorrhage.   Therefore, based on the clinical presentation and characteristic features, the provisional diagnosis in this case is subarachnoid hemorrhage. Urgent medical attention is necessary to confirm the diagnosis and begin appropriate treatment.


27. A 14-year-old boy developed a sympatho-adrenal crisis. What medicine was indicated for crisis relief in this case?

A. Propranolol

B. Digoxin

C. Thiopental sodium

D. Euphyllin (Aminophylline)

E. Drotaverine


Answer: Propranolol

Explanation

The sympatho-adrenal crisis is characterized by excessive activation of the sympathetic nervous system and increased secretion of catecholamines, such as epinephrine and norepinephrine. The symptoms of a sympatho-adrenal crisis can include hypertension, tachycardia, headache, sweating, and anxiety.

The medication that is indicated for crisis relief in this case is propranolol. Propranolol is a beta-blocker that works by blocking the effects of catecholamines on the sympathetic nervous system. It can help to reduce blood pressure, heart rate, and other symptoms associated with a sympatho-adrenal crisis. Propranolol can also be used to prevent future episodes of crisis in patients with conditions such as pheochromocytoma or neuroblastoma.

Digoxin is a medication used to treat heart failure and certain arrhythmias, but it is not indicated for the treatment of a sympatho-adrenal crisis. Thiopental sodium is a short-acting barbiturate that is used as an anesthetic agent and for the treatment of status epilepticus, but it is not indicated for the treatment of a sympatho-adrenal crisis.

Euphyllin (Aminophylline) is a medication that is used to treat respiratory conditions such as asthma and chronic obstructive pulmonary disease (COPD) by relaxing the smooth muscles of the airways. It is not indicated for the treatment of a sympatho-adrenal crisis.

Drotaverine is a medication that is used to treat smooth muscle spasms, such as those that occur in the gastrointestinal tract or in the uterus during labor. It is not indicated for the treatment of a sympatho-adrenal crisis. Therefore, based on the clinical presentation and the appropriate medication for crisis relief, the correct answer is propranolol.

28. In the morning, an 8-year-old child started complaining of general weakness and pain during swallowing. At noon, her parents called for a doctor, because the girl’s weakness was progressing and she developed a bilateral swelling slightly below and in front of her ears. Make the right diagnosis:

A. Mumps

B. Lymphadenopathy

C. Tumor of the salivary glands

D. Dermoid cysts

E. Tumor of the carotid body


Answer: Mumps

Explanation

The clinical presentation of an 8-year-old child with general weakness, pain during swallowing, and swelling below and in front of the ears is most consistent with mumps.   Mumps is a viral infection that affects the salivary glands, causing swelling and tenderness below and in front of the ears.

The infection is usually accompanied by symptoms such as fever, headache, muscle aches, and fatigue. Pain during swallowing can also occur due to the inflammation of the salivary glands.   Lymphadenopathy, or the enlargement of lymph nodes, can also cause swelling below and in front of the ears, but is not typically associated with general weakness and pain during swallowing. Lymphadenopathy can be caused by a variety of conditions, including infections, autoimmune diseases, and malignancies.  

Tumors of the salivary glands or carotid body can also cause swelling in the area below and in front of the ears, but are less common in children and typically do not present with general weakness and pain during swallowing. Dermoid cysts are also a possibility, but are typically painless and do not cause systemic symptoms.  

Therefore, based on the clinical presentation and the most likely diagnosis, the correct answer is mumps. It is important to confirm the diagnosis with laboratory tests and provide appropriate treatment, as mumps can lead to complications such as meningitis, encephalitis, and orchitis.


29. A 58-year-old patient, a heavy drinker and smoker, came to a hospital with complaints of constant coughing and shortness of breath. Lately, he has been losing weight. His cervical lymph nodes are enlarged and dense, the tissues above them exhibit no tension. Chest X-ray has shown fibrosis of an upper pulmonary lobe and left-sided pleurisy. The pleural fluid is straw-colored, with protein levels of 52 g/L and a high lymphocyte count. Malignant cells were not detected. Inoculation of the pleural fluid produced no microbial growth one week later. Make the right diagnosis:

A. Pulmonary tuberculosis

B. Systemic lupus erythematosus

C. Sarcoidosis

D. Bronchiectasis

E. Atypical pneumonia


Answer: Pulmonary tuberculosis

Explanation

The clinical presentation of a 58-year-old patient who is a heavy drinker and smoker, with constant coughing, shortness of breath, weight loss, enlarged cervical lymph nodes, fibrosis of an upper pulmonary lobe, and left-sided pleurisy with straw-colored pleural fluid containing high levels of protein and lymphocytes, but no malignant cells or microbial growth, is highly suggestive of pulmonary tuberculosis.  

Tuberculosis (TB) is a bacterial infection caused by the Mycobacterium tuberculosis bacteria. It primarily affects the lungs but can also spread to other parts of the body. The symptoms of TB can include coughing, shortness of breath, weight loss, and enlarged lymph nodes. Pulmonary fibrosis and pleurisy can also occur as a result of the infection.  

The presence of straw-colored pleural fluid with high protein levels and a high lymphocyte count is consistent with a lymphocytic exudative effusion, which can occur in TB and other infectious diseases, as well as in non-infectious conditions such as malignancy and autoimmune disorders. However, the absence of malignant cells and microbial growth, along with the clinical presentation, make TB the most likely diagnosis.  

Systemic lupus erythematosus (SLE), sarcoidosis, bronchiectasis, and atypical pneumonia can also cause respiratory symptoms and pleural effusion, but are less likely given the clinical presentation and pleural fluid characteristics.   Therefore, based on the clinical presentation and the most likely diagnosis, the correct answer is pulmonary tuberculosis.

Further diagnostic tests, such as sputum culture and nucleic acid amplification tests, may be necessary to confirm the diagnosis and determine the appropriate treatment.


30. A laboratory has received a milk sample for analysis. The findings are as follows: color is whitish, smell is normal, taste is normal for milk, specific gravity is 1,038, acidity is 35oT (degrees Turner), fats is 3.2%. What is the milk quality? 922521I1 5

A. The milk is of poor quality

B. The milk is of reduced quality

C. The milk is conditionally fit for use

D. The milk is a counterfeit product

E. The milk is of good quality


Answer:  The milk is of poor quality

Explanation

The laboratory findings of the milk sample suggest that the milk is of poor quality.   The color, smell, and taste of the milk are reported as normal, but these characteristics alone do not indicate the overall quality of the milk.

The specific gravity, which is a measure of the density of the milk, is reported as 1.038, which is slightly higher than the normal range of 1.027 to 1.033 for cow’s milk. This may indicate that the milk has a higher concentration of solids, which could affect its quality.  

The acidity of the milk is reported as 35oT, which is above the normal range of 15 to 20oT for fresh milk. High acidity can indicate that the milk has started to sour or spoil, which can reduce its quality and make it unsafe for consumption.  

The fat content of the milk is reported as 3.2%, which is within the normal range for whole milk. However, the fat content alone does not determine the overall quality of the milk.   Based on the laboratory findings, it can be concluded that the milk is of poor quality, as it has high acidity and a slightly elevated specific gravity.

This could be due to factors such as bacterial contamination, poor handling and storage, or adulteration with water or other substances.   Therefore, the correct answer is A. The milk is of poor quality. It is not conditionally fit for use, nor is it a counterfeit product.


31. A 7-year-old child fell ill 2 weeks ago, when he developed a runny nose. An otolaryngologist was consulted about sanguinopurulent discharge from the child’s nose and maceration of his wings of the nose and upper lip. Rhinoscopy has detected whitish-gray foci on the nasal septum. The oropharyngeal mucosa is without changes. Make the right diagnosis:

A. Nasal diphtheria

B. Adenovirus infection

C. Maxillary sinusitis

D. Rhinovirus infection

E. Allergic rhinitis


Answer:  Nasal diphtheria

Explanation

The clinical presentation of a 7-year-old child with sanguinopurulent nasal discharge, maceration of the wings of the nose and upper lip, and whitish-gray foci on the nasal septum is highly suggestive of nasal diphtheria.  

Nasal diphtheria is a bacterial infection caused by Corynebacterium diphtheriae. It is characterized by a thick, grayish-white membrane that forms on the nasal mucosa, which can lead to airway obstruction and difficulty breathing. The maceration and sanguinopurulent discharge can occur as a result of tissue damage and inflammation caused by the infection.  

Adenovirus infection, rhinovirus infection, and allergic rhinitis can also cause nasal discharge and inflammation, but are typically not associated with the formation of a grayish-white membrane on the nasal mucosa.  

Maxillary sinusitis can also cause nasal discharge and inflammation, but is typically accompanied by facial pain and tenderness, and may be associated with fever and malaise.   Therefore, based on the clinical presentation and the most likely diagnosis, the correct answer is nasal diphtheria. It is important to confirm the diagnosis with laboratory tests and provide appropriate treatment, as diphtheria can lead to serious complications such as respiratory failure, myocarditis, and neuropathy.


32. An 18-year-old young man complains of pain in his knee and ankle joints and a fever of 39.5oC. A week and a half before, he had a case of respiratory disease. His body temperature is 38.5oC, his knee and ankle joints are swollen. His pulse is 106/min., rhythmic. His blood pressure is 90/60 mm Hg. The heart borders are normal, the heart sounds are weakened, and there is a soft systolic murmur at the apex. What parameter was most closely associated with the possible etiology of this process?

A. Antistreptolysin 0

B. Seromucoid

C. Creatine kinase

D. Rheumatoid factor

E. α1-antitrypsin


Answer: Antistreptolysin 0

Explanation

The clinical presentation of an 18-year-old young man with joint pain, fever, and a recent history of respiratory disease is suggestive of rheumatic fever, which is a complication of untreated or inadequately treated streptococcal pharyngitis.   The presence of joint pain and swelling, along with fever, suggests an inflammatory process that affects multiple joints.

The soft systolic murmur at the apex of the heart and weakened heart sounds suggest involvement of the heart, which is a common complication of rheumatic fever. The association with a recent respiratory infection further supports the possibility of rheumatic fever.  

Antistreptolysin O (ASO) is an antibody that is produced in response to infection with streptococcal bacteria. ASO levels can be measured in the blood and are used to diagnose a recent or ongoing streptococcal infection. Elevated ASO levels are also a hallmark of rheumatic fever, as the immune response to the streptococcal infection can lead to cross-reactivity with the tissues of the joints, heart, and other organs.  

Seromucoid, creatine kinase, α1-antitrypsin, and rheumatoid factor are not closely associated with the etiology of rheumatic fever and would not typically be used to diagnose or monitor the condition.   Therefore, based on the clinical presentation and the most likely diagnosis, the parameter that is most closely associated with the possible etiology of this process is antistreptolysin O (ASO). A blood test to measure ASO levels would be useful in confirming the diagnosis of rheumatic fever and guiding appropriate treatment.


33. A postparturient woman came to a doctor on the 14th day after giving birth. She complained of a sudden pain, hyperemia, and induration in her left breast, fever of 39oC, headache, and indisposition. There is a fissure in the nipple, the left breast is enlarged, during its palpation the pain intensifies. What pathology is suspected in this case?

A. Lactational mastitis

B. Breast cancer

C. Suppurated cyst of the left breast

D. Phlegmon of the breast

E. Fibroadenoma of the left breast


Answer: Lactational mastitis

Explanation

The clinical presentation of a postparturient woman with sudden pain, hyperemia, and induration in the left breast, fever, headache, and indisposition, along with a fissure in the nipple and pain during palpation, is highly suggestive of lactational mastitis.   Lactational mastitis is an inflammatory condition of the breast tissue that occurs during lactation.

It is often associated with a blocked milk duct, which can cause milk to build up and lead to infection. The symptoms of lactational mastitis can include breast pain, redness, swelling, and warmth, along with fever and flu-like symptoms such as headache and malaise.  

Breast cancer, suppurated cyst of the breast, phlegmon of the breast, and fibroadenoma of the breast can also cause breast pain and swelling, but are less likely given the clinical presentation and the fact that the patient recently gave birth.  

Therefore, based on the clinical presentation and the most likely diagnosis, the correct answer is lactational mastitis. Treatment may include antibiotics to treat the infection and continued breastfeeding or milk expression to alleviate the blockage. It is important to promptly treat lactational mastitis to prevent complications such as abscess formation.


34. A 15-year-old patient presents with delayed physical development and periodically develops icteric skin. Spleen is 16x12x10 cm, cholecystolithiasis is observed in the patient, there is a skin ulcer on the left calf in its lower third. The blood has: erythrocytes – 3.0 · 1012/L, Hb – 90 g/L, color index – 1.0, microspherocytosis, reticulocytosis. Total serum bilirubin – 56 mcmol/L, indirect bilirubin – 38 mcmol/L. What treatment method is optimal in this case?

A. Splenectomy

B. Omentohepatopexy

C. Spleen transplant

D. Omentosplenopexy

E. Portocaval anastomosis

Answer: Splenectomy


Explanation

The clinical presentation of a patient with a color index of 1.0, microspherocytosis, reticulocytosis, and elevated levels of total serum bilirubin and indirect bilirubin suggests hemolytic anemia, which is a condition characterized by the destruction of red blood cells and subsequent elevation of bilirubin levels.  

Splenectomy, or surgical removal of the spleen, is a treatment option for certain types of hemolytic anemia. The spleen plays a role in removing damaged or abnormal red blood cells from circulation, so its removal can reduce the destruction of red blood cells and lower bilirubin levels.  

Omentohepatopexy, spleen transplant, omentosplenopexy, and portocaval anastomosis are not typically used as treatment options for hemolytic anemia.   Therefore, based on the clinical presentation and the most appropriate treatment option, the optimal treatment method in this case is splenectomy.

However, the decision to proceed with splenectomy should be made on a case-by-case basis, taking into account the underlying cause of the hemolytic anemia and other factors such as the patient’s overall health and medical history.


35. A 22-year-old woman at 11–12 weeks of pregnancy came to a maternity clinic. Examination has shown a positive Wasserman reaction. A dermatologist diagnosed her with secondary latent syphilis. What are the tactics of pregnancy management in such case?

A. Artificial termination of the pregnancy after a course of antisyphilitic therapy

B. Prolongation of the pregnancy after a course of antisyphilitic therapy

C. Artificial termination of the pregnancy after the diagnosis is made

D. Artificial termination of the pregnancy after normalization of the patient’s Wasserman reaction

E. Three antisyphilitic treatment courses throughout the pregnancy


Answer:Artificial termination of the pregnancy after a course of antisyphilitic therapy 

Explanation

The presence of a positive Wasserman reaction and a diagnosis of secondary latent syphilis in a pregnant woman requires careful management to prevent transmission of the infection to the fetus. The optimal approach in this case would be to provide a course of antisyphilitic therapy to the woman, which can effectively treat the infection and reduce the risk of transmission to the fetus.

Treatment with benzathine penicillin G is the recommended therapy for syphilis during pregnancy. After completion of the course of antisyphilitic therapy, the pregnancy should be terminated artificially. This is necessary to prevent transmission of the infection to the fetus, as syphilis can cause serious complications such as stillbirth, prematurity, and congenital syphilis.

Prolongation of the pregnancy after a course of antisyphilitic therapy is not recommended in cases of syphilis during pregnancy, as the risk of transmission to the fetus remains high. Artificial termination of the pregnancy after the diagnosis is made or after normalization of the patient’s Wasserman reaction is not the recommended approach, as these factors alone do not ensure that the infection has been effectively treated and the risk of transmission to the fetus has been eliminated.

Three antisyphilitic treatment courses throughout the pregnancy are not necessary and may expose the fetus to unnecessary risks associated with multiple courses of antibiotics.   Therefore, based on the clinical presentation and the most appropriate management approach, the correct answer is artificial termination of the pregnancy after a course of antisyphilitic therapy.


36. A lumbar puncture was made for a newborn with a suspected intracranial birth injury. Bloody cerebrospinal fluid was obtained. What type of hemorrhage was observed in this case?

A. Subarachnoid hemorrhage

B. Cephalohematoma

C. Subtentorial hemorrhage

D. Epidural hemorrhage

E. Supratentorial hemorrhage


Answer:  Subarachnoid hemorrhage

Explanation

The presence of bloody cerebrospinal fluid obtained by lumbar puncture in a newborn with suspected intracranial birth injury suggests a subarachnoid hemorrhage.   Subarachnoid hemorrhage is a type of bleeding that occurs between the arachnoid membrane and the pia mater, which are the layers of tissue that surround the brain and spinal cord. In newborns, it can be caused by a traumatic birth injury, such as a difficult delivery or the use of forceps or vacuum extraction.  

Cephalohematoma is a collection of blood under the scalp of a newborn, which is typically caused by a birth injury such as pressure against the mother’s pelvis during delivery. It does not involve bleeding into the cerebrospinal fluid.   Subtentorial hemorrhage and supratentorial hemorrhage refer to bleeding within specific areas of the brain, and are not specific to newborns or birth injuries.   Epidural hemorrhage is a rare type of bleeding that occurs between the skull and the dura mater, which is the outermost layer of tissue that surrounds the brain.

It is typically caused by trauma to the head or skull fracture, and is not associated with newborns or birth injuries.   Therefore, based on the clinical presentation and the most likely diagnosis, the type of hemorrhage observed in this case is subarachnoid hemorrhage, which requires appropriate medical management and monitoring to prevent complications such as seizures, hydrocephalus, and developmental delays.


37. The mother of a newborn child is suffering from chronic pyelonephritis and had a case of acute respiratory viral infection before giving birth. The delivery is full-term, the waters did not break for a long time. On the 2nd day of life the baby developed an erythematous rash. Blisters of about 1 cm in size appeared. They are filled with seropurulent contents and their lancing results in formation of erosions. The Nikolsky’s sign is positive. The baby is inert and has a subfebrile body temperature. Make the diagnosis:

A. Neonatal pemphigus

B. Vesiculopustulosis

C. Sepsis

D. Ritter’s disease of the newborn

E. Pseudofurunculosis


Answer: Neonatal pemphigus

Explanation

The clinical presentation of an erythematous rash with blisters filled with seropurulent contents, positive Nikolsky’s sign, and the development of erosions in a newborn, along with a history of chronic pyelonephritis and an acute respiratory viral infection in the mother, is highly suggestive of neonatal pemphigus.  

Neonatal pemphigus is a rare autoimmune disorder that can occur in infants born to mothers with autoimmune diseases, such as chronic pyelonephritis. The disorder is characterized by the development of blisters and erosions on the skin and mucous membranes, which can be severe and may lead to scarring and other complications.  

Vesiculopustulosis, Ritter’s disease of the newborn, and pseudofurunculosis are other skin conditions that can occur in newborns, but they do not typically present with the same clinical features as neonatal pemphigus.  

Sepsis is a serious and potentially life-threatening infection that can occur in newborns, but it typically presents with other signs and symptoms such as fever, lethargy, poor feeding, and respiratory distress, which are not specific to neonatal pemphigus.   Therefore, based on the clinical presentation and the most likely diagnosis, the correct answer is neonatal pemphigus. Treatment typically involves the use of systemic corticosteroids and other immunosuppressive medications, along with supportive care and management of any associated complications.


38. A 34-year-old patient has been suffering from pulmonary tuberculosis for the last 7 years. He complains of muscle weakness, weight loss, diarrhea, and frequent urination. His skin, gums, and buccal mucosa are hyperpigmented. His blood pressure is 90/58 mm Hg. In the blood: erythrocytes – 3.1 · 1012/L, Hb – 95 g/L, color index – 0.92; leukocytes – 9.4 · 109/L, eosinophils – 7%, segmented neutrophils – 45%, band neutrophils – 1%, lymphocytes – 40%, monocytes – 7%, Na+ – 115 mmol/L, + – 7.3 mmol/L. Make the diagnosis:

A. Primary insufficiency of the adrenal cortex

B. Primary hyperaldosteronism

C. Congenital hyperplasia of the adrenal cortex

D. Pheochromocytoma

E. Diabetes insipidus


Answer: Primary insufficiency of the adrenal cortex

Explanation

The clinical presentation of a patient with a history of pulmonary tuberculosis and symptoms such as muscle weakness, weight loss, hyperpigmentation of the skin, gums, and buccal mucosa, and electrolyte imbalances such as low sodium and high potassium levels, along with abnormalities in blood cell counts, is highly suggestive of primary insufficiency of the adrenal cortex, also known as Addison’s disease.  

Addison’s disease is a condition in which the adrenal glands do not produce enough of the hormones cortisol and aldosterone, which are necessary for regulating metabolism, blood pressure, and electrolyte balance. It can be caused by autoimmune disorders, infections such as tuberculosis, or genetic factors.  

Primary hyperaldosteronism, congenital hyperplasia of the adrenal cortex, and pheochromocytoma are other adrenal conditions that can cause electrolyte imbalances and other symptoms, but are less likely given the clinical presentation and history of pulmonary tuberculosis.   Diabetes insipidus is a condition characterized by excessive thirst and urination due to a deficiency of the hormone vasopressin, which is produced by the hypothalamus and regulates the balance of fluids in the body.

It is not typically associated with adrenal insufficiency.   Therefore, based on the clinical presentation and the most likely diagnosis, the correct answer is primary insufficiency of the adrenal cortex, or Addison’s disease. Treatment typically involves replacement of the deficient hormones with corticosteroids and mineralocorticoids, along with management of any associated complications.


39. A 35-year-old woman has red swollen areas on the dorsal surface of her hands after a severe nervous strain, which was followed by formation of small inflamed nodules, vesicles, and later erosions with significant discharge of a serous fluid. This process was accompanied by severe itching. Make the right diagnosis:

A. True eczema

B. Microbial eczema

C. Toxicoderma

D. Simple contact dermatitis

E. Allergic dermatitis


Answer: True eczema

Explanation

Based on the clinical presentation of red swollen areas on the dorsal surface of the hands followed by the formation of small inflamed nodules, vesicles, and later erosions with severe itching, the most likely diagnosis is true eczema, also known as atopic dermatitis.  

True eczema is a chronic inflammatory skin condition characterized by itching and recurrent episodes of flares and remissions. It can be triggered by a variety of factors, including stress, irritants, allergens, and infections.   Microbial eczema, also known as infected eczema, is a type of eczema that is caused by a bacterial infection. It typically presents with symptoms such as crusting, oozing, and pain, which are not described in the case presented.  

Toxicoderma is a type of skin reaction that occurs as a result of exposure to a drug or chemical. While it can present with symptoms similar to those of eczema, such as redness, itching, and vesicles, it is not typically associated with a history of severe nervous strain.  

Simple contact dermatitis and allergic dermatitis are other types of skin reactions that can present with symptoms similar to those of eczema, but they are typically localized to the area of skin that came into contact with the irritant or allergen, whereas eczema can affect multiple areas of the body.  

Therefore, based on the clinical presentation and the most likely diagnosis, the correct answer is true eczema, or atopic dermatitis. Treatment typically involves the use of topical corticosteroids, moisturizers, and other medications to control itching and inflammation, as well as identification and avoidance of triggers to prevent flares.


40. A 48-year-old patient, whose blood test has resulted in positive Wasseman and sedimentation reactions thrice, came to a doctor. His treponema pallidum immobilization test was positive as well. Examination has shown no clinical manifestations of syphilis in the patient’s skin, internal organs or nervous system. Make the right diagnosis:

A. Early latent syphilis

B. Latent recurrent syphilis

C. Tertiary syphilis

D. Seropositive primary syphilis

E. Recurrent secondary syphilis


Answer: Early latent syphilis

Explanation

Based on the clinical presentation and laboratory results, the most likely diagnosis is early latent syphilis.   Syphilis is a sexually transmitted infection caused by the bacterium Treponema pallidum. It can progress through several stages, including primary, secondary, latent, and tertiary syphilis.  

During the latent stage, which can last for years, there are no clinical manifestations of syphilis. However, laboratory tests such as the Wasseman and sedimentation reactions, as well as the treponema pallidum immobilization test, can be positive.  

Early latent syphilis refers to the period of syphilis that occurs within the first year after initial infection, during which there are no clinical manifestations of the disease but serologic tests are positive. In this case, the positive laboratory tests suggest that the patient has syphilis, but there are no clinical signs or symptoms of the disease in the skin, internal organs, or nervous system.  

Latent recurrent syphilis, recurrent secondary syphilis, and seropositive primary syphilis are other stages and forms of syphilis that are less likely based on the clinical presentation and laboratory results.  

Tertiary syphilis is a late stage of the disease that can occur many years after initial infection, and is characterized by the development of serious complications such as neurosyphilis, cardiovascular syphilis, and gummatous syphilis.   Therefore, based on the clinical presentation and laboratory results, the most likely diagnosis is early latent syphilis.

Treatment typically involves a course of antibiotics such as penicillin to eliminate the infection and prevent progression to more serious stages of the disease.


41. A 50-year-old man was hospitalized with complaints of blood in the urine. There are no pain or urination disorders. Hematuria is observed for the last 3 days. The kidneys are not palpable, the suprapubic area is normal, the external genitalia have no pathology. Rectal examination detects no prostatic enlargement. Cystoscopy detects no changes. What disease is suspected first?

A. Kidney cancer

B. Renal dystopia

C. Necrotizing papillitis

D. Varicocele

E. Tuberculosis of the urinary bladder


Answer: Kidney cancer

Explanation

In a 50-year-old man with complaints of painless hematuria for the last 3 days, and with no other symptoms or physical findings, the most likely diagnosis is kidney cancer.   Hematuria, or blood in the urine, is a common symptom of kidney cancer. In some cases, the bleeding may be visible, as in the case described, while in other cases it may only be detected on laboratory testing.

Kidney cancer is more common in men than in women and typically affects people over the age of 40.   Renal dystopia, or ectopic kidney, is a congenital condition in which the kidney is located in an abnormal position, but it typically does not cause hematuria.  

Necrotizing papillitis is a condition characterized by inflammation and necrosis of the renal papillae, which can cause hematuria and pain, but typically presents with other symptoms such as fever and flank pain, which are not described in the case presented.  

Varicocele is a condition in which the veins in the scrotum become enlarged and twisted, causing pain and swelling, but it typically does not cause hematuria.   Tuberculosis of the urinary bladder can cause hematuria and other urinary symptoms, but it typically presents with other symptoms such as fever, weight loss, and fatigue, which are not described in the case presented.  

Therefore, based on the clinical presentation and the most likely diagnosis, the correct answer is kidney cancer. Further diagnostic tests such as imaging studies (e.g. CT or MRI) and biopsy may be necessary to confirm the diagnosis and determine the extent of the disease. Treatment typically involves surgery to remove the affected kidney, along with other therapies such as radiation and chemotherapy depending on the stage and severity of the cancer.


42. A 28-year-old patient complains of chills in his feet and fingers and pain in his legs during walking. The disease onset was one year ago. The skin of his legs is pale and cold; low turgor and hypotrichosis are observed. Pulsation over the femoral and popliteal arteries is weakened, on the pedal arteries it is palpable after a nitroglycerin test. Rheographic index is <1. Make the right diagnosis:

A. Obliterating endarteritis

B. Chronic thrombophlebitis

C. Wegener vasculitis

D. Arteriosclerosis obliterans

E. Raynaud syndrome


Answer: Obliterating endarteritis

Explanation

Based on the clinical presentation of chills in the feet and fingers, pain in the legs during walking, pale and cold skin of the legs, low turgor, hypotrichosis, weakened pulsation over the femoral and popliteal arteries, and rheographic index <1, the most likely diagnosis is obliterating endarteritis, also known as peripheral arterial disease (PAD).  

PAD is a condition in which the arteries that supply blood to the legs become narrowed or blocked, leading to reduced blood flow and oxygen delivery to the affected tissues. This can cause symptoms such as pain, cramping, and numbness in the legs during activity, as well as changes in the skin and nails, such as pale or bluish discoloration, thinning of the skin, and decreased hair growth.  

Chronic thrombophlebitis is a condition in which there is inflammation and clotting of the veins, which can cause pain, swelling, and redness in the affected area. This condition is less likely given the presentation of arterial involvement in this case.  

Wegener vasculitis is a rare autoimmune disease that affects the blood vessels, causing inflammation and damage to the walls of the vessels. It typically presents with symptoms such as fever, weight loss, joint pain, and skin rash, which are not described in the case presented.  

Arteriosclerosis obliterans is a type of PAD that is caused by the buildup of fatty deposits in the arteries. It presents with symptoms similar to those of obliterating endarteritis, but the term specifically refers to atherosclerosis of the lower extremities.  

Raynaud syndrome is a condition in which the blood vessels in the fingers and toes spasm and constrict, causing numbness, pain, and color changes in the affected area. It typically presents with symptoms that affect the hands and feet specifically, which is not described in the case presented.  

Therefore, based on the clinical presentation and the most likely diagnosis, the correct answer is obliterating endarteritis, or peripheral arterial disease. Treatment typically involves lifestyle modifications such as exercise and smoking cessation, medications to improve blood flow and reduce symptoms, and in severe cases, surgical or endovascular interventions to restore blood flow to the affected area.


43. A 20-year-old woman complains of amenorrhea. She has hirsutism and obesity with predominant distribution of adipose tissue on the face, neck, and upper torso. On her face she has acne vulgaris, on her skin there are stretch marks. The blood pressure is 170/100 mm Hg. Her mental development and intelligence are not impaired. Gynecological status shows moderate hairiness of the external genitalia and acute hypoplasia of the vagina and uterus. Make the right diagnosis:

A. Cushing syndrome

B. Stein-Leventhal syndrome

C. Turner syndrome

D. Sheehan syndrome

E. Babinski-Froehlich syndrome


Answer: Cushing syndrome

Explanation

Based on the clinical presentation of amenorrhea, hirsutism, obesity with predominant distribution of adipose tissue on the face, neck, and upper torso, acne vulgaris, stretch marks, high blood pressure, moderate hairiness of the external genitalia, and acute hypoplasia of the vagina and uterus, the most likely diagnosis is Cushing syndrome.  

Cushing syndrome is a disorder that occurs when the body is exposed to high levels of cortisol over an extended period of time, either due to excess production of cortisol by the adrenal glands or prolonged use of glucocorticoid medications. Symptoms of Cushing syndrome include weight gain, particularly around the face, neck, and upper torso, which is described in this case as well as hypertension, acne, and hirsutism.

Amenorrhea may also occur due to disruption of the normal menstrual cycle.   Stein-Leventhal syndrome, also known as polycystic ovary syndrome (PCOS), is a condition characterized by multiple cysts on the ovaries, menstrual irregularities, and signs of hyperandrogenism such as hirsutism and acne. However, obesity and hypertension are not typically associated with PCOS.  

Turner syndrome is a genetic disorder that affects females, and is characterized by short stature, primary amenorrhea, and abnormalities of the reproductive system such as ovarian dysgenesis. However, obesity, hirsutism, and hypertension are not typical features of Turner syndrome.  

Sheehan syndrome is a condition that occurs due to ischemic necrosis of the pituitary gland, typically following severe bleeding during childbirth. It can cause hypopituitarism and secondary amenorrhea, but is not typically associated with obesity, hirsutism, or hypertension.  

Babinski-Froehlich syndrome is not a recognized medical condition.   Therefore, based on the clinical presentation and the most likely diagnosis, the correct answer is Cushing syndrome. Treatment typically involves addressing the underlying cause of excess cortisol production, such as surgery or medication adjustments, as well as managing symptoms such as hypertension and insulin resistance.


44. A 48-year-old patient was hospitalized with complaints of headache, nausea, wet cough, problematic breathing, impaired vision, excessive sweating, and salivation. He worked in a team that treated gardens with organophosphorus pesticides. In his blood: erythrocytes – 4.1 · 1012/L, Hb – 136 g/L, color index – 0.9, leukocytes – 13.0 · 109/L, ESR – 17 mm/hour. He was diagnosed with acute intoxication caused by organophosphorus pesticides. What is the most important diagnostic criterion o 922521I1 7 this pathology?

A. Decreased cholinesterase levels

B. Reticulocytosis

C. Leukocytosis

D. Thrombocytopenia

E. Anemia


Answer: Decreased cholinesterase levels

Explanation

The most important diagnostic criterion for acute intoxication caused by organophosphorus pesticides is decreased cholinesterase levels.   Organophosphorus pesticides work by inhibiting the activity of cholinesterase, an enzyme that breaks down the neurotransmitter acetylcholine.


When cholinesterase activity is inhibited, acetylcholine builds up in the body, leading to overstimulation of the nervous system and other organs.   Symptoms of acute intoxication with organophosphorus pesticides can include headache, nausea, vomiting, cough, respiratory distress, visual disturbances, excessive sweating, salivation, and muscle twitching or weakness. The severity of symptoms depends on the level and duration of exposure.  

Diagnosis of acute organophosphorus pesticide poisoning is based on a combination of clinical symptoms and laboratory tests. The most important diagnostic criterion is decreased cholinesterase levels, which can be measured in blood or other tissues. Cholinesterase levels can be used to assess the severity of poisoning and monitor the effectiveness of treatment.  

Reticulocytosis, leukocytosis, thrombocytopenia, and anemia may occur in some cases of acute organophosphorus pesticide poisoning, but they are not specific diagnostic criteria and may also occur in other conditions.  

Therefore, based on the clinical presentation and laboratory results, the most important diagnostic criterion for acute intoxication caused by organophosphorus pesticides is decreased cholinesterase levels. Treatment typically involves removing the individual from further exposure to the pesticide, administering antidotes such as atropine and pralidoxime, and providing supportive care such as oxygen therapy and mechanical ventilation if necessary.


45. A 55-year-old man on the 3rd day after an acute anterior myocardial infarction complains of shortness of breath and a dull pain behind the sternum that decreases when he leans forward. His blood pressure is 140/80 mm Hg and his heart sounds are muffled. ECG has shown atrial fibrillation with the ventricular rate of 110/min., pathological Q wave, and ST segment elevation in the right-sided chest leads. Make the right diagnosis:

A. Acute pericarditis

B. Dressler syndrome

C. Tietze syndrome

D. Pulmonary embolism

E. Dissecting aortic aneurysm


Answer: Acute pericarditis

Explanation

The most likely diagnosis for this patient is acute pericarditis.   Acute pericarditis is inflammation of the pericardium, the thin sac surrounding the heart. It can occur after a myocardial infarction and is characterized by chest pain, which is often described as sharp or stabbing and worsens with inspiration, coughing, or lying down, and improves with leaning forward.

Other symptoms can include shortness of breath and muffled heart sounds.   The presence of atrial fibrillation with a rapid ventricular response, pathological Q waves, and ST segment elevation in the right-sided chest leads are all consistent with a recent myocardial infarction.

However, the patient’s symptoms of chest pain that improves with leaning forward and muffled heart sounds are more suggestive of pericarditis.   Dressler syndrome is a rare complication of myocardial infarction that occurs weeks to months after the initial event, and is characterized by pericarditis, pleuritis, and fever.

This patient’s symptoms are more consistent with acute pericarditis occurring shortly after the myocardial infarction.   Tietze syndrome is a condition characterized by inflammation of the costochondral junctions, which can cause chest pain and swelling, but it is not typically associated with muffled heart sounds or shortness of breath.  

Pulmonary embolism is a blockage of one or more pulmonary arteries in the lungs, which can cause chest pain, shortness of breath, and other symptoms, but it is not typically associated with muffled heart sounds or chest pain that improves with leaning forward.  

Dissecting aortic aneurysm is a medical emergency in which there is a tear in the wall of the aorta, which can cause severe chest pain, but it is not typically associated with muffled heart sounds or chest pain that improves with leaning forward.  

Therefore, based on the clinical presentation and ECG findings, the most likely diagnosis for this patient is acute pericarditis. Treatment typically involves anti-inflammatory medications such as aspirin or nonsteroidal anti-inflammatory drugs (NSAIDs), and in some cases, colchicine or corticosteroids, along with treatment of any underlying conditions such as myocardial infarction or atrial fibrillation.


46. The blood pressure and age were studied among 200 patients with essential hypertension. What statistical value can be used to measure the strength of the relationship between these characteristics?

A. Correlation coefficient

B. Representation error

C. Variation coefficient

D. Sigma deviation

E. Student’s t-test


Answer:  Correlation coefficient

Explanation

The statistical value that can be used to measure the strength of the relationship between blood pressure and age in 200 patients with essential hypertension is the correlation coefficient.   Correlation coefficient is a statistical measure that indicates the degree of association or relationship between two variables. I

n this case, the two variables of interest are blood pressure and age. The correlation coefficient ranges from -1 to +1, with values closer to -1 indicating a negative correlation (inverse relationship), values closer to +1 indicating a positive correlation (direct relationship), and values closer to 0 indicating no correlation.  

A positive correlation between blood pressure and age would suggest that as age increases, blood pressure tends to increase as well, while a negative correlation would suggest the opposite. A correlation coefficient of 0 would indicate that there is no relationship between blood pressure and age.  

Representation error, variation coefficient, sigma deviation, and Student’s t-test are not appropriate statistical measures for assessing the relationship between blood pressure and age.   Representation error is a measure of how well a sample represents the population from which it was drawn, and is not directly related to the relationship between two variables.   Variation coefficient measures the relative variability of a dataset, and is not specifically designed to measure the relationship between two variables.  

Sigma deviation is a measure of the dispersion or spread of a dataset, and is also not specifically designed to measure the relationship between two variables.   Student’s t-test is a statistical test used to determine if there is a significant difference between the means of two groups, and is not designed to measure the correlation or relationship between two variables.  

Therefore, the most appropriate statistical value to measure the strength of the relationship between blood pressure and age in 200 patients with essential hypertension is the correlation coefficient.


47. A 38-year-old woman, para 3, has a history of 5 artificial abortions. Five minutes after giving birth, she started bleeding from her genital tracts (350 mL). The patient’s condition is satisfactory, her pulse is 92/min., blood pressure is 100/60 mm Hg. There are no signs of placental expulsion, its manual removal has to be performed. In some spots the placenta is somewhat difficult to detach. Make the right diagnosis:

A. Partial placenta adherens

B. Trapped placenta

C. Total placenta adherens

D. Placenta accreta

E. Hypotonic bleeding


Answer:Partial placenta adherens

Explanation

The most likely diagnosis for this patient is partial placenta adherens.   Placenta adherens refers to a condition in which the placenta does not separate from the uterine wall after delivery, and can result in postpartum hemorrhage. In partial placenta adherens, only a portion of the placenta remains attached to the uterine wall, while in total placenta adherens, the entire placenta remains attached.   In this case, the patient experienced bleeding from the genital tract shortly after giving birth, which is suggestive of postpartum hemorrhage.

The fact that the placenta did not spontaneously detach and had to be manually removed also suggests a placental adherence disorder. The difficulty in detaching some spots of the placenta further supports the diagnosis of partial placenta adherens.   Trapped placenta is a condition in which the placenta is retained within the uterus after delivery, and can also result in postpartum hemorrhage. However, the fact that the placenta was manually removed suggests that it was not simply trapped.  

Placenta accreta is a condition in which the placenta grows too deeply into the uterine wall, and can result in difficulty separating the placenta after delivery. However, the fact that only some spots of the placenta were difficult to detach suggests partial placenta adherens rather than placenta accreta.   Hypotonic bleeding is a type of postpartum hemorrhage that occurs due to inadequate uterine contraction, and is not directly related to placental adherence disorders.   Therefore, based on the clinical presentation and the need for manual removal of the placenta with difficulty detaching some spots, the most likely diagnosis for this patient is partial placenta adherens. Treatment typically involves manual removal of the placenta, followed by monitoring for signs of postpartum hemorrhage and management of any associated complications.


48. A woman, who works at a pig farm, against the background of complete health developed sudden chills, fever of 39.9oC, intense headache, and nausea. The next day, she noticed a pain in her leg muscles and nosebleeds. On the 3rd day after the onset of the disease the woman’s condition is severe, her face is hyperemic, she has scleritis and subicteric sclerae. The liver is +3 cm. 24-hour urine output is 700 mL. Make the right diagnosis:

A. Leptospirosis

B. Yersiniosis

C. Influenza

D. Viral hepatitis

E. Hemorrhagic fever with renal syndrome


Answer: Leptospirosis

Explanation

The symptoms of sudden chills, fever, headache, nausea, and muscle pain along with nosebleeds are indicative of a bacterial infection, and the presence of scleritis and subicteric sclerae suggest a possible liver involvement. The enlarged liver and low urine output suggest impaired kidney function.

These signs and symptoms are consistent with leptospirosis, a bacterial infection caused by the spirochete Leptospira interrogans, which is commonly associated with exposure to contaminated water or soil, as is often seen in individuals working on pig farms.  

Yersiniosis can also cause fever, headache, and muscle pain, but it is typically associated with gastrointestinal symptoms such as diarrhea and abdominal pain, which are absent in this case. Influenza can cause similar symptoms, but it typically resolves within a few days and does not cause liver or kidney involvement. Viral hepatitis can cause liver enlargement and yellowing of the skin and eyes (jaundice), but it does not typically cause nosebleeds or scleritis.


Hemorrhagic fever with renal syndrome (HFRS) is caused by a group of viruses such as Hantavirus, which can also cause fever, headache, and muscle pain, but it is typically associated with kidney involvement and hemorrhagic symptoms, which are not prominent in this case.   Therefore, the most likely diagnosis in this case is Leptospirosis. A prompt diagnosis and treatment with antibiotics are necessary to prevent complications such as kidney failure and meningitis.


49. A 38-year-old man lives in an area that is endemic for echinococcosis. For the last 6 months he has been suffering from pain in his right subcostal region and fever. Echinococcal liver damage was suspected. What study will be the most informative in this case?

A. Ultrasound examination

B. Liver scan

C. Angiography

D. Biochemical testing

E. Survey X-ray-of the abdominal cavity


Answer: Ultrasound examination

Explanation

In a patient with suspected echinococcal liver damage, the most informative study would be an ultrasound examination. Echinococcosis, also known as hydatid disease, is a parasitic infection caused by the larval stage of the tapeworm Echinococcus granulosus.

The liver is the most commonly affected organ, and the presence of a cyst in the liver is highly suggestive of echinococcal disease.   Ultrasound examination is a non-invasive, widely available, and relatively inexpensive imaging modality that can detect echinococcal cysts with a high degree of sensitivity and specificity. It can also provide information about the size, location, and number of cysts, as well as the presence of complications such as rupture or infection.  

Liver scan, angiography, biochemical testing, and survey X-ray of the abdominal cavity are less informative in the diagnosis of echinococcal liver damage. A liver scan may show areas of decreased or increased activity, but it does not provide detailed information about the cystic nature of the lesion.

Angiography may show the presence of vascular abnormalities, but it does not provide information about the cystic nature of the lesion either. Biochemical testing may reveal elevated liver function tests, but it is not specific for echinococcosis. Survey X-ray of the abdominal cavity can detect calcified cysts, but it is less sensitive than ultrasound examination for detecting early-stage or small cysts.   In summary, ultrasound examination is the most informative in the diagnosis of echinococcal liver damage.

50. A 29-year-old woman was brought to the admission room with complaints of a sudden pain in her lower abdomen, weakness, and loss of consciousness. Her last menstruation was 6 weeks ago. Her pulse is 120/min., blood pressure – 80/50 mm Hg. There are pain and signs of peritoneal irritation low on the right side. In the blood: Hb – 106 g/L. Make the right diagnosis:

A. Interrupted tubal pregnancy

B. Ovarian torsion

C. Ovarian apoplexy

D. Acute appendicitis

E. Acute right-sided adnexitis


Answer:Interrupted tubal pregnancy

Explanation

The sudden onset of lower abdominal pain, weakness, and loss of consciousness in a woman who is 6 weeks late for her period, along with signs of peritoneal irritation and low blood pressure, suggests a ruptured ectopic pregnancy. The presence of pain and signs of peritoneal irritation low on the right side further suggest a tubal pregnancy.

The low hemoglobin level may be indicative of internal bleeding. Therefore, the right diagnosis in this case is Interrupted tubal pregnancy.   Ovarian torsion and ovarian apoplexy can cause sudden onset of lower abdominal pain, but they do not typically cause loss of consciousness or signs of peritoneal irritation.


Acute appendicitis typically causes pain in the right lower quadrant, but it does not usually cause loss of consciousness or peritoneal irritation. Acute right-sided adnexitis can cause lower abdominal pain on the right side, but it is not typically associated with loss of consciousness or signs of peritoneal irritation.  

An interrupted tubal pregnancy is a medical emergency and requires prompt management. The patient should be stabilized with intravenous fluids and blood transfusion, if necessary. Surgery is usually required to remove the ectopic pregnancy and repair any damage to the fallopian tube. If left untreated, a ruptured ectopic pregnancy can lead to life-threatening complications such as hemorrhage and shock.

Join the conversation
0% Complete